Как вычислить Ri & Ra лампы

McPaul

Местный
Уважаемые товарищи. Помогите советом или формулой как расчитать внутреннее сопротивление лампы с целью подобрать оптимальную нагрузку.
Дело в том что столкнулся с такой ситуацией, сделал драйвер на лампе 6Н3П, а R ее не знаю, поставил методом научного тыка 22 к на каждую половинку.
А хотелось бы большей гибкости для варьирования параметрами каскада.
 

Lex

Member
Уважаемые товарищи. Помогите советом или формулой как расчитать внутреннее сопротивление лампы с целью подобрать оптимальную нагрузку.
Дело в том что столкнулся с такой ситуацией, сделал драйвер на лампе 6Н3П, а R ее не знаю, поставил методом научного тыка 22 к на каждую половинку.
А хотелось бы большей гибкости для варьирования параметрами каскада.

*** Вы хотели наверно спросить как найти Ri лампы? Потому как Ra это приведенное сопротивление нагрузки в случае с трансформаторным каскадом. Ri берется из справочников либо обмер лампы. Может изменяться в некоторых пределах в зависимости от выбора рабочей точки. Ri=дельтаU/дельтаI на ВАХе лампы. Кажется так...
 

Iosaaris

Местный
*** Вы хотели наверно спросить как найти Ri лампы?
Мне кажется, что вопрос был как выбрать сопротивление анодной нагрузки в обычном резистивном каскаде, как в примере с 6Н3П, который привёл МакПауль.
 

Lex

Member
Мне кажется, что вопрос был как выбрать сопротивление анодной нагрузки в обычном резистивном каскаде, как в примере с 6Н3П, который привёл МакПауль.

***Возможно и так. В таком случае нужно просто взять в руки РАСЧЕТ каскада на резисторах и посчитать.
 

Pavel

Member
Бывают случаи, когда в справочнике не приведено внутренне сопротивление, тогда его можно рассчитать по формуле Ri=mu/S.
Можно определить Ri графически по ВАХ. Для этого через выбранную рабочую точку проводят касательную к линии напряжения смещения на сетке. По углу наклона этой касательной рассчитывается Ri=дельтаU/дельта I.
Выбор Ra - это совсем другая песня. Из букварей следует, что Ra надо выбирать в пределах (2-5)Ri. В резистивном каскаде выбор Ra определяется конкретными условиями (требованиями): допустимыми искажениями, напряжением источника питания, входной емкостью лампы следующего каскада (хотя, это больше относится к выбру лампы по амплитуде выходного тока, чем к выбору Ra).
 

McPaul

Местный
Во! я имел ввиду именно как вычислить внутреннее сопротивление лампы, поскольку на многие лампы в справочниках таких данных нет.
С остальными лампами, например там 6н2 проще, поскольку на ней много кто делал схемы, и там уже просто тупо берешь и повторяешь номиналы, что в свою очередь не хорошо.
А вот как расчитать сопротивление нагрузки - это уже так сказать поле для экспериментов.
 

Iosaaris

Местный
Можно определить Ri графически по ВАХ. Для этого через выбранную рабочую точку проводят касательную к линии напряжения смещения на сетке. По углу наклона этой касательной рассчитывается Ri=дельтаU/дельта I.
Павел, посмотрите аттач. Вы это имели в виду? Если я вас правильно понял, то в данном примере берём на синей прямой две точки. Для удобства возьмём точку 120В 40мА и точку 320В 60мА. Считаем: (320-120)/(60-40)=200/20=10К. Я правильно понял?
 

Вложения

  • 6p14p_2z.jpg
    74.1 KB · Просмотры: 244

Zef

Местный
Ri это наклон ветки ВАХ там, где она вырождается в прямую. Ra не вычисляется, а прикидывается на ВАХ при помощи линейки.

Есть 3 типа триодов:
- с "параллельными" ВАХ. Для таких Ra чем больше - тем лучше, идеал - SRPP, минимум - перпендикуляр к ВАХ.
- С "веерными" ВАХ. Ra - перпендикуляр к ВАХ в рабочей точке.
- С криволинейными ВАХ. Возим линеечкой и меряем расстояние между точками пересечения так, чтобы добиться наилучшей линейности.
 

Pavel

Member
2 Iosaaris
Да, именно это я и имел ввиду. Вот только пример получился не очень удачный. Согласитесь, Ri=10 кОм - не очень пентодное сопротивление - получено потому, что рабочая точка находится на нелинейном участке ВАХ.
Тут прав Zef, и если р.т. взять на прямом (насколько это возможно) участке, то и Ri возрастет до 30-40 кОм (навскидку), что уже похоже на правду для пентода.
 

Iosaaris

Местный
Вот только пример получился не очень удачный. Согласитесь, Ri=10 кОм - не очень пентодное сопротивление - получено потому, что рабочая точка находится на нелинейном участке ВАХ.
Мне цифра в 10К тоже показалась странноватой, но эту точку я выбрал специально для того, чтобы можно было изобразить касательную, а не линию, которая полностью наложится на сам ВАХ.

Ra не вычисляется, а прикидывается на ВАХ при помощи линейки.

Есть 3 типа триодов:
- С "веерными" ВАХ. Ra - перпендикуляр к ВАХ в рабочей точке.
- С криволинейными ВАХ. Возим линеечкой и меряем расстояние между точками пересечения так, чтобы добиться наилучшей линейности.
А как из перпендикуляра к ВАХ вычислить само сопротивление Ra?
 

Pavel

Member
По наклону нагрузочной прямой, точно так же, как и Ri, Ra=дельтаU/дельтаI
 

Iosaaris

Местный
Кстати, а оптимальное сопротивление анодной нагрузки тоже можно по ВАХам вычислить?
 

Pavel

Member
Оптимальное по какому показателю?
По мощности, отдаваемой в нагрузку, или по уровню искажений - можно.
Для этого рассчитывают приведенные выше характеристики для разных наклонов нагрузочной прямой.
 

Iosaaris

Местный
А как я могу знать при каком наклоне нагрузочной прямой будет максимальная мощность, а при каком минимальные искажения?
 

Pavel

Member
Путем расчета.
Пример:
Выбираете значения Ra, равные, например, 3Ri, 5Ri и 8Ri. Для каждого значения наносите на ВАХ соответствующую нагрузочную прямую. Для каждого значения Ra (нагрузочной прямой) графическим способом рассчитываете мощность, отдаваемую в нагрузку (первичную обмотку трансформатора) и Коэффициент гармонических искажений.
Далее по результатам выбираете наилучшее для вас значение Ra.
Конечно, и без всех этих хлопот ясно, что (в общем случае) чем меньше Ra, тем больше мощность и искажения, и наоборот. Однако зависят эти показатели от величины нагрузки непропорционально, поэтому сделать сознателный компромиссный выбор достаточного (и реализуемого) значения Ra без рассчетов невозможно.
 

Гэгэн

Active member
А как я могу знать при каком наклоне нагрузочной прямой будет максимальная мощность, а при каком минимальные искажения?

Максимальная мощность, отдаваемая триодом при Ra=2Ri,
Минимальный КНИ нужно расчитывать не только по 2й гармонике, по ней получается чем больше Ra-тем меньше КНИ, желательно расчитывать и по 3й гармонике оптимальную нагрузку.
 

booch

Местный
Максимальная мощность, отдаваемая триодом при Ra=2Ri
Вероятно описка. В усилителе класа А вроде максимальная мощность получается при Ra=Ri. Ra=2Ri обычно рекомендуется как минимальный компромис между мощностью и нелинейными искажениями.
 

Fil

Местный
Да нет, все правильно Ra=2Ri - максимальная мощность, а все что выше - компромисы. Rвых=Rнагр это немного из другой оперы, это из ТОЭ и линейных цепей (коими лампы не являются).
 

Iosaaris

Местный
Максимальная мощность, отдаваемая триодом при Ra=2Ri,
А у пентода?

Минимальный КНИ нужно расчитывать не только по 2й гармонике, по ней получается чем больше Ra-тем меньше КНИ, желательно расчитывать и по 3й гармонике оптимальную нагрузку.
А как расчитывается КНИ и гармоники? Или хотя бы где об этом можно прочитать?
 
Сверху